Difference between revisions of "Talk:2011 AMC 12B Problems/Problem 9"

(Created page with "Is the problem flawed? The range of numbers [-20, 10] includes 31 total numbers: [-1, -20], 0, and [1, 10]. Which makes the probabilities drastically different. For example the p...")
 
 
Line 1: Line 1:
Is the problem flawed? The range of numbers [-20, 10] includes 31 total numbers: [-1, -20], 0, and [1, 10]. Which makes the probabilities drastically different. For example the probability for two consecutive negative numbers being chosen becomes <math>\frac{20}{31} \cdot \frac{20}{31} = \frac{400}{961}</math>
+
Is the problem flawed? The range of numbers [-20, 10] includes 31 total numbers: [-1, -20], 0, and [1, 10]. Which makes the probabilities drastically different as we're searching for a product that is greater than zero. For example the probability for two consecutive negative numbers being chosen becomes <math>\frac{20}{31} \cdot \frac{20}{31} = \frac{400}{961}</math>

Latest revision as of 01:39, 21 December 2011

Is the problem flawed? The range of numbers [-20, 10] includes 31 total numbers: [-1, -20], 0, and [1, 10]. Which makes the probabilities drastically different as we're searching for a product that is greater than zero. For example the probability for two consecutive negative numbers being chosen becomes $\frac{20}{31} \cdot \frac{20}{31} = \frac{400}{961}$